Water flows from a bathroom tap at a rate of 2 gallons every 6 seconds. At this rate, how many minutes will it take to fill an 80-gallon tub?

Answers

Answer 1

Answer:

240 minutes

Step-by-step explanation:

i divide 80 divided by 2 then multiply the answer which is 40 by 6 and get 240


Related Questions

Evaluate -3.28 - (-4.4) + (-p) where p = 9.7

Answers

Answer:

-8.58

Step-by-step explanation:

p = 9.7

=> -3.28 - (-4.4) + (-p)

=> -3.28 - (-4.4) + (- (9.7))

=> -3.28 - (-4.4) + (-9.7)

=> -3.28 - (-4.4) -9.7

=> -3.28 +4.4 -9.7

=> -12.98 +4.4

=> -8.58

Answer:

The evaluated answer for this equation is -8.58

Step-by-step explanation:

We are given a value of p so we will use this value for the equation.

-3.28 - (-4.4) + (-p)

Replace p with 9.7

-3.28 - (-4.4) + (-9.7)

Since there is a double negative in front of 4.4, then that means that we are going to be adding -3.28 to 4.4

1.12 + (-9.7)

Subtract 1.42 from 9.7 because there is negative sign in front of 9.7

-8.58

1. Which expression is equivalent to (-2)(a + 6)?

Answers

Answer:

please mark my answer brainliest

Step-by-step explanation:

- 2a -12

Simplify the expression. (3x2 – 4x + 1) + (-x2 + x – 9)

Answers

[tex](3x^2 - 4x + 1) + (-x^2 + x - 9)=\\3x^2-4x+1-x^2+x-9=\\2x^2-3x-8[/tex]

= (6x+4x+1)+(-2x+x-9)
= (2x+1)+(-x-9)
= 2x+1-x-9
= x-8

Consider f(x) = 3x2 + 4 and g(x) = 2x − 3. Which statements about f + g are true? Select all that apply. A. The domain is all real numbers. B. The range is all real numbers. C. It is a linear function. D. It is a quadratic function.

PLEASE HELP

Answers

Answer:

B. The range is all real numbers. D. It is a quadratic function.

Step-by-step explanation:

Given the functions  f(x) =  3x² + 4 and g(x) = 2x − 3, to know that statements that is  true about f+g, we will need to add the functions together first.

f(x)+g(x) = 3x² + 4 + 2x - 3

f(x)+g(x) = 3x²+2x +4 - 3

f(x)+g(x)  = 3x²+2x + 1

The highest degree of a quadratic function is 2 and since the highest degree of the resulting function is 2, hence the resulting sum of the equations is quadratic.

Also the range of the values is all real numbers because the presence of x² in the function will always return a positive value greater than x.

Hence the correct statements are 'the range is all real numbers and it is a quadratic function'

Answer:

B AND D

Step-by-step explanation:

What is the value of z for the equation fraction 1 over 2z = −fraction 3 over 4 + fraction 1 over 4z? −3 −1 1 3

Answers

Answer:

z= -3

Step-by-step explanation:

Given:

1/2z =-3/4 + 1/4z

Collect like terms

1/2z - 1/4z = -3/4

Add 1/2z - 1/4z

2z-z / 4 = -3/4

We have

z/4=-3/4

Same as

z(1/4) = -3/4

Divide both sides by 1/4

z(1/4) ÷ 1/4 = -3/4÷1/4

z(1/4) × 4/1= -3/4 × 4/1

z(4/4) = -12/4

z= -3

The value of z= -3

Answer:

-3

Step-by-step explanation:

I got it right on the test

Which of the following lists of three numbers could form the side lengths of a triangle? A. 10, 20, 30 B. 122, 257, 137 C. 8.6, 12.2, 2.7 D. 1/2, 1/5, 1/6

Answers

Answer:

Step-by-step explanation:

The triangle inequality theorem states that the sum of any two sides of a triangle os greater than the third side.

■■■■■■■■■■■■■■■■■■■■■■■■■■

First triangle:

Let a,b and c be the sides of the triangle:

● a = 10

● b = 20

● c = 30

Now let's apply the theorem.

● a+b = 10+20=30

That's equal to the third side (c=30)

●b+c = 50

That's greater than a.

● a+c = 40

That's greater than b.

These aren't the sides of a triangel since the first inequality isn't verified.

■■■■■■■■■■■■■■■■■■■■■■■■■

Second triangle:

● a = 122

● b = 257

● c = 137

Let's apply the theorem.

● a+b = 379

That's greater than c

● a+c = 259

That's greater than b

● b+c = 394

That's greater than a

So 122,257 and 137 can be sides of a triangle.

■■■■■■■■■■■■■■■■■■■■■■■■■■

The third triangle:

● a = 8.6

● b = 12.2

● c = 2.7

Let's apply the theorem:

● a+b = 20.8

That's greater than c

● b+c = 14.9

That's greater than a

● a+c = 11.3

That isn't greater than b

So theses sides aren't the sides of triangle.

■■■■■■■■■■■■■■■■■■■■■■■■■■

● a = 1/2

● b = 1/5

● c = 1/6

Let's apply the theorem.

● a+b = 7/10

That's greater than c

● a+c = 2/3

That's greater than b

● b+c = 11/30

That isn't greater than a

So these can't be the sides of a triangle.

PLEASE HELPP on THIS PICTURE FOR ONE OF MY QUESTIONS

Answers

Answer:

Linear pair postulate

Step-by-step explanation:

The Linear Pair Postulate states: "If two angles form a linear pair, then the angles are supplementary; that is, the sum of their measures is 180 degrees

A linear pair of angles is such that the sum of angles is 180 degrees.

Given the right triangle below, if AB = 4 and BC = 4, find AC.
A
B
C

Answers

The length of AC is found using the Pythagorean theorem which is a^2+b^2=c^2.

In this case your equation would look like 4^2+4^2=c^2.
Four to the power of two is 16 so 16+16=c^2.
16+16=32
32=c^2
Root 32 = c

Length AC is root 32.

AC will be 4√2 when AB = 4 and BC = 4, in the given right triangle.

What is Pythagoras' Theorem?

According to Pythagoras' Theorem, in a right triangle, the square of the length of the longest side, that is, the hypotenuse, that is, the side opposite to the right angle is equal to the sum of the squares of the lengths of the other two sides.

How to solve the question?

In the question, we are given a right triangle, with sides AB = 4 and BC = 4.

We are asked to find AC.

To find AC, we will use the Pythagoras theorem, according to which, we can write:

AC² = AB² + BC²

or, AC² = 4² + 4²,

or, AC² = 16 + 16,

or, AC² = 32,

or, AC = √32,

or, AC = √(16 * 2) = 4√2.

Therefore, AC will be 4√2 when AB = 4 and BC = 4, in the given right triangle.

Learn more about Pythagoras' Theorem at

https://brainly.com/question/231802

#SPJ2

Which expression is equivalent to (–2)(a + 6)?
A. –2a + 6
B. 2a + 12
C. –2a – 12
D. –2a + 12

Answers

The answer is option c.

What property is demonstrated here? (3x-5) x 4 = 3 x (-5 x 4) A) commutative property of addition B) associative property of multiplication C) commutative property of multiplication D) associative property of addition (haven't learned this yet so I have no clue)

Answers

Answer:

B) Associative Property of Multiplication

Step-by-step explanation:

*if it's wrong idk how, but I apologise*

0,2,4,0,2,3,2,8,6

What is the mean?
What is the median?!
What is the first quartile (Q1)?!
What is the third quartile (Q3)?
What is the minimum?
What is the maximum?
What is the interquartile range of the data?!

Answers

Answer:

LOOK BELOW

Step-by-step explanation:

Mean= add all of the numbers together and divide by total amount of

            numbers

            aka=3

Median= put all the numbers in order and find the number in the middle

               aka=2.5

Minimum=0

Maximum=8

This data can only be found using a Box and Whisker Plot....

I can't really explain a box and whisker plot so you have to look that up to understand that..  SRY!!!

------------------------------------

First Quartile=1

Third Quartile=5

Interquartile Range=4

Answer:

Mean= add all of the numbers together and divide by total amount of

           numbers

           aka=3

Median= put all the numbers in order and find the number in the middle

              aka=2.5

Minimum=0

Maximum=8

This data can only be found using a Box and Whisker Plot....

I can't really explain a box and whisker plot so you have to look that up to understand that..  SRY!!!

------------------------------------

First Quartile=1

Third Quartile=5

Interquartile Range=4

Step-by-step explanation:

solve for x 5(x+1)=4(x+8)

Answers

Answer:

x=27

Step-by-step explanation:

expanding the above expression we get

5x+5=4x+32

grouping numbers with coefficient of x at the left side and constant at the right side we get

5x-4x=32-5

x=27

To solve this equation, we start by distributing both the 5 and the 4 through both set of parentheses.

This gives us 5x + 5 = 4x + 32.

Now subtract 4x from both
sides to get x + 5 = 32.

Now subtract 5 from both sides and x = 27.

If you invest $600 at 5% interest compounded continuously, how much would you make after 6 years?

Answers

Answer:

809.915$

Step-by-step explanation:

Amount of money = Principal x e^(rate x year)

                              = 600 x e^(0.05 x 6)

                              = 809.915$

Answer:

$809.92

Step-by-step explanation:

(see attached for reference)

Recall that the formula for compound interest (compounded continuously) is

A = P e^(rt)

where,

A = final amount (we are asked to find this)

P = principal = given as $600

r = interest rate = 5% = 0.05

t = time = 6 years

e = 2.71828 (mathematical constant)

Substituting the known values into the equation:

A = P e^(rt)

= 600 e^(0.05 x 6)

= 600 (2.71828)^(0.30)

= $809.92

What is another way to write 100,203 in other forms

Answers

Answer:

You can write in

Step-by-step explanation:

Lakhs

First write 100,203.

Put ones,tens,hundreds,thousands,ten thousands and lakh on top of each number from the extreme left.

This is how you can write 100,203 in another way. You can't write in any other way than this one.

Hope this helps....

Have a nice day!!!!

Marta esta poniendo sus libros en una estantería. Le faltan 7 libros para poder poner 12 en cada estante; sin embargo, si pone 10 libros en cada estante, se quedan 5 libros sin poner. ¿Cuantos es antes tiene la estantería?

Answers

Answer:

x = 6       la cantidad de estantes

y = 65    cantidad de libros

Step-by-step explanation:

LLamemos "x" la cantidad de estantes que tiene Marta, y llamaremos "y" la cantidad de libros.

La primera condición que se debe cumplir es que cuando Marta coloca 12 libros en cada estante entonces le faltan 7, esto lo expresamos así:

y  +  7  = 12*x        (1)

La segunda condición establece que si Marta coloca los libros en número de 10 por estante le quedan 5 sin colocar, luego esto en lenguaje matemático se expresa así:

y  -   5   = 10*x     (2)

Ahora hemos obtenido un sistema de dos ecuaciones con dos incógnitas que se resuelve por cualquiera de los métodos conocidos, usaremos el método de sustitución.

Despejamos   y en la primera ecuación y lo sustituimos en la segunda, de esa forma obtendremos el valor de x

y  =  12*x  - 7

(12*x - 7 ) - 5  = 10*x

2*x  -12 = 0

2*x = 12

x = 6       la cantidad de estantes, y

y  =  12*x -7

y  =  72  -  7

y =  65    cantidad de libros

Please answer quickly i will give you brainliest if its correct- it has to be a simplified fraction please

Answers

Answer:

[tex]\large \boxed{{r=\frac{1}{9}}}[/tex]

Step-by-step explanation:

x and y are proportional.

[tex]y=rx[/tex]

Let x = 45 and y = 5.

[tex]5=r(45)[/tex]

Solve for r (constant of proportionality).

Divide both sides by 45.

[tex]\displaystyle \frac{5}{45} =r[/tex]

Simplify and switch sides.

[tex]\displaystyle r=\frac{1}{9}[/tex]

Which of the following statements about fractions is not true? a. Proper fractions have a greater numerator than denominator b. Improper fractions are percentages greater than 100% c. Mixed fractions can be written as improper fractions d. The product of a fraction and its reciprocal is 1

Answers

Answer:

c

Step-by-step explanation:

Answer:

A. Proper fractions have a greater numerator than denominator.

what is the radical of 5√72 PLZ HELP!

Answers

Answer: Exact Form: 30√2

Decimal Form:42.42640687…

Step-by-step explanation: Simplify the radical by breaking the radicand up into a product of known factors, assuming positive real numbers.

I hope this helped :)

Answer:

  30√2

Step-by-step explanation:

The radical portion of the given expression is √72.

__

Perhaps you want the simplest form of your expression. Factor out the perfect squares from under the radical.

  [tex]5\sqrt{72}=5\sqrt{36}\sqrt{2}=5\cdot 6\sqrt{2}=\boxed{30\sqrt{2}}[/tex]

Allison is rolling her hula hoop on the playground. The radius of her hula hoop is 35 \text{ cm}35 cm35, start text, space, c, m, end text. What is the distance the hula hoop rolls in 444 full rotations?

Answers

Answer: 880 cm

Step-by-step explanation:

Given: Radius of the hula hoop = 35 cm

Hula hoop is  circular in shape

Then, Circumference = [tex]2\pi r[/tex] , where r = radius

Now , Circumference of hula hoop = [tex]2\times \dfrac{22}{7}\times35=220\ cm[/tex]

Now , the distance the hula hoop rolls in 4 full rotations = 4 × (Circumference of hula hoop)

[tex]= 4 \times 220=880\ cm[/tex]

Hence, the required distance = 880 cm

Answer:

880

Step-by-step explanation:

There are a total of two hundred students and chaperones going on a
field trip. Each bus can hold 60 passengers. How many buses will be
used for the field trip? Explain why your answer is reasonable.

Answers

Answer:

4 bus is required for field trip to carry 200 passengers.

Step-by-step explanation:

Total no . of passengers = 200

let the be x bus required to carry 200 passengers

capacity of 1 bus = 60 passengers

capacity of x bus = 60*x passengers = 60x passengers

Thus,

60x = 200

x = 200/60 = 3 2/3

thus, 3.66 bus is required , but no. of bus cannot be in fraction hence we take integral value greater than 3.66 which is 4

Thus, 4 bus is required for field trip to carry 200 passenger.

The row-echelon form of the augmented matrix of a system of equations is given.Find the solution of the system

Answers

Answer:

x = 9/4

y = 3/5

z = 2/3

w = -9/5

Step-by-step explanation:

Technically, the matrix is in reduced row echelon form. If there are zeros above and below the ones, it is RREF. If there are zeros only below the ones, then it's REF.

Since it is in RREF, the augmented numbers to the right of the bar are already your solutions. Simply label the variables.

asap!!
~~~~~~
A line passes through point (–6, –1) and is parallel to the equation y = –2x – 5. What's the equation of the line?

Question 25 options:

y = –2x – 13

y = 12{"version":"1.1","math":"\(\frac{1}{2}\)"}x + 3

y = –12{"version":"1.1","math":"\(\frac{1}{2}\)"}x – 1

y = 2x + 5

click on picture for a, b, c ,or d

Answers

Answer:

y=−2x−13.

Step-by-step explanation:

The equation of the line in the slope-intercept form is y=−2x−5.

The slope of the parallel line is the same: m=−2.

So, the equation of the parallel line is y=−2x+a.

To find a, we use the fact that the line should pass through the given point: −1=(−2)⋅(−6)+a.

Thus, a=−13.

Therefore, the equation of the line is y=−2x−13.

find the interest rate r when p = 800, a = 2700, and t = 3.​

Answers

Answer:

r = 0.5 or 1/2

Step-by-step explanation:

Simple Interest Rate Formula: A = P(1 + r)^t

Simply plug in our known variables:

2700 = 800(1 + r)³

Now we solve for r:

Divide both sides by 800

27/8 = (1 + r)³

Take the cube root on both sides

∛27/8 = ∛(1 + r)³

Simplify

3/2 = 1 + r

Subtract 1 on both sides

r = 1/2

r = 0.5

Answer:

For compound interest, 50%.

Step-by-step explanation:

(I'm assuming this question is asking for the compound interest):

The formula for compound interest is given by:

[tex]A=P(1+\frac{r}{n})^{nt}[/tex]

Plug in the values we know. We can use 1 for n:

[tex]2700=800(1+r)^3\\27/8=(1+r)^3\\1+r=\sqrt[3]{27/8}\\r=3/2-1\\r=1/2=.5[/tex]

So, the interest rate is 50%.

help..? why are there so many parentheses..?can you plz give a step by step on how to slove the equation?

Answers

Answer:

= -11

Step-by-step explanation:

-(-(11-22))

= -(-11+22)

= 11 - 22

= -11

A car is averaging 50 miles per hour. If the car maintains this speed, how many minutes less would a 450-mile trip take than a 475-mile trip?

Answers

Answer:

1/2 a minute (30 seconds)

Step-by-step explanation:

475/50=9.5

450/50=9

9-9.5=.5

Solve logs (8 - 3x) = log20 for x.
A. X = 14
B. X = -13
C.x = -8
D. X= -4

Answers

Answer:

x = -4

Step-by-step explanation:

logs (8 - 3x) = log20

Since we are taking the log on each side

log a = log b  then a = b

8 -3x = 20

Subtract 8  from each side

8 -3x-8 =20 -8

-3x = 12

Divide by -3

-3x/-3 = 12/-3

x = -4

Answer:

[tex] \boxed{\sf x = -4} [/tex]

Step-by-step explanation:

[tex] \sf Solve \: for \: x \: over \: the \: r eal \: numbers:[/tex]

[tex] \sf \implies log(8 - 3x) = log 20[/tex]

[tex] \sf Cancel \: logarithms \: by \: taking \: exp \: of \: both \: sides:[/tex]

[tex] \sf \implies 8 - 3x = 20[/tex]

[tex] \sf Subtract \: 8 \: from \: both \: sides:[/tex]

[tex] \sf \implies 8 - 3x - 8 = 20 - 8 [/tex]

[tex] \sf \implies - 3x = 12 [/tex]

[tex] \sf Divide \: both \: sides \: by \: - 3:[/tex]

[tex] \sf \implies \frac{-3x}{-3} = \frac{12}{-3} [/tex]

[tex] \sf \implies x = - 4[/tex]

1A-MATH
Diviwuetalls
2 Exercise 7:
Your ansv
Type you
The three angles in a triangle are labeled A, B and C.
A = 2x
B= 3x
C=4x
Private coi
1) calculate X
2) find the actual angle of B
3) find the exterior angle to B

Answers

Answer:

Value of x = 20°

Angle B = 60°

Exterior angle to B = 300°

Step-by-step explanation:

Given:

Angles of triangle.

A = 2x

B = 3x

C = 4x

We know that,

A + B + C = 180°

2x + 3x + 4x = 180°

9x = 180°

x = 20°

Value of x = 20°

Angle B = 3x

Angle B = 3(20°)

Angle B = 60°

Exterior angle to B = 360° - 60°

Exterior angle to B = 300°

**Yoxelt buys 4 1/ 2 gallons of soda. One-fourth of the soda he bought was Pepsi and the rest was Sprite. How many gallons of Pepsi did Yoxelt buy? Show all work below.

Answers

Answer:

1 1/8

Step-by-step explanation:

1/4 of 4 1/2 is Pepsi.

1/4 * 4 1/2 = (1/4) * 4 + (1/4) * (1/2) = 1 1/8

Using a table of values, determine the solution to the equation below to the nearest fourth of a unit. 2^x=1-3^x

Answers

Answer:

Option (1)

Step-by-step explanation:

Given equation is,

[tex]2^x=1-3^x[/tex]

To determine the solution of the equation we will substitute the values of 'x' given in the options,

Option (1)

For x = -0.75

[tex]2^{-0.75}=1-3^{-0.75}[/tex]

0.59 = 1 - 0.44

0.59 = 0.56

Since, values on both the sides are approximately same.

Therefore, x = -0.75 will be the answer.

Option (2)

For x = -1.25

[tex]2^{-1.25}=1-3^{-1.25}[/tex]

0.42 = 1 - 0.25

0.42 = 0.75

Which is not true.

Therefore, x = -1.25 is not the answer.

Option (3)

For x = 0.75

[tex]2^{0.75}=1-3^{0.75}[/tex]

1.68 = 1 - 2.28

1.68 = -1.28

Which is not true.

Therefore, x = 0.75 is not the answer.

Option (4)

For x = 1.25

[tex]2^{1.25}=1-3^{1.25}[/tex]

2.38 = 1 - 3.95

2.38 = -2.95

It's not true.

Therefore, x = 1.25 is not the answer.

What number must be added to the expression for it to equal zero? (–6.89 + 14.52) + (–14.52)

Answers

Answer:

The number to be added is 6.89

Step-by-step explanation:

Here, we want to know what number must be added to the expression to make it equal to zero.

Let the number be x

Thus;

-6.89 + 14.52 -14.52 + x = 0

-6.89 + x = 0

x = 6.89

Other Questions
The following information is available for Armstrong Company: Net income $450 Increase in plant and equip. $170 Depreciation expense 80 Payment of dividends 10 Decrease in accts. receiv. 20 Increase in long-term debt 100 Increase in inventories 15 Decrease in accounts payable 30 What is cash flow from operating activities for Armstrong Company? what is the value of -19- (-18)? Capitalism gets its name from the fact that capital resources:_________ a. are mostly treated as private property. b. owned by the state or government. c. given the highest priority in the economy's income distribution. d. in the form of money and financial resources. Ava placed the point of her pencil on the origin of a regular coordinate plane. She marked a point after moving her pencil 4 units to the left and 7 units up. Which ordered pair identifies where Ava marked her point? Even though most corporate bonds in the United States make coupon payments semiannually, bonds issued elsewhere often have annual coupon payments. Suppose a German company issues a bond with a par value of 1,000, 15 years to maturity, and a coupon rate of 7.3 percent paid annually.If the yield to maturity is 8.6 percent, what is the current price of the bond? (Do not round intermediate calculations and round your final answer to 2 decimal places. (e.g., 32.16))Current price Sunny Day Manufacturing Company is considering investing in a one-year project that requires an initial investment of $450,000. To do so, it will have to issue new common stock and will incur a flotation cost of 2.00%. At the end of the year, the project is expected to produce a cash inflow of $550,000. The rate of return that Sunny Day expects to earn on its project (net of its flotation costs) is:____________White Lion Homebuilders has a current stock price of $22.35 per share, and is expected to pay a per-share dividend of $2.03 at the end of next year. The company's earnings' and dividends' growth rate are expected to grow at the constant rate of 8.70% into the foreseeable future. If White Lion expects to incur flotation costs of 5.00% of the value of its newly-raised equity funds, then the flotation-adjusted (net) cost of its new common stock (rounded to two decimal places) should be:_________Sunny Day Manufacturing Company Co.'s addition to earnings for this year is expected to be $420,000. Its target capital structure consists of 50% debt, 5% preferred, and 45% equity. Determine Sunny Day Manufacturing Company's retained earnings breakpoint: ___________a. $840,000 b. $980,000 c. $933,333 d. $886,666 Look at the figure below. which ratio represents tan 0?A -5/4, B -4/5, C -3/4, D 3/5. In the Andes Mountains of South America, entire towns of thousands of people have been buried suddenly when water and loose materials traveled downhill as a viscous fluid. This type of catastrophic event is known as a Find the sine and cosine of each angle as a fraction and as a decimal. Round to the nearest hundredth. 1. Suppose that a solid ball, a solid disk, and a hoop all have the same mass and the same radius. Each object is set rolling without slipping up an incline with the same initial linear (translational) speed. Which goes farthest upthe incline?a. the ballb. the diskc. the hoopd. the hoop and the disk roll to the same height, fartherthan the balle. they all roll to the same height2. Suppose that a solid ball, a solid disk, and a hoop all have the same mass and the same radius. Each object is set rolling with slipping up an incline with the same initial linear (translational) speed. Which goes farthest upthe incline?a. the ballb. the diskc. the hoopd. the hoop and the disk roll to the same height, fartherthan the balle. they all roll to the same height The table shows the probability distribution of student ages in a high schoolwith 1500 students. What is the expected value for the age of a randomlychosen student?Age131415161718Probability 0.01 0.23 0.26 0.28 0.20 0.02 Could anyone help me with this question please? Thank you. Nursing is a Woman's JobPlease discuss Find the measure of a. The receipt of dividends and interest from abroad as a result of ownership of foreign assets by a country's residents is recorded as what:____________. Question 38(Multiple Choice Worth 2 points)[OSS.01]The theories surrounding the formation of our solar system are based on manybiased opinionsincorrect factsnon-testable datascientific investigations a pupil performed an experiment in a school lab to show the action of a digestive enzyme on a food substancea) Name and enzyme suitable for such an experiment b) Name a food substance on which the enzyme that you have named will actc) Describe any preparation of the food required before the experiment is performed. If no preparation is required state why? d) give the temperature at which the enzyme-food mix should be maintained for the experiment to work e) how much time is needed for digestion of food in this experiment? f) describe a test to confirm that digestion has occurred Which of the following was considered the capital of the Eastern Roman Empire? A. Anatolia B. Constantinople C. Rome D. Macedonia For which system of inequalities is (3,-7) a solution? A. x + y < -4 3x + 2y < -5 B. x + y -4 3x + 2y < -5 C. x + y < -4 3x + 2y -5 D. x + y -4 3x + 2y -5 What is the solution set for StartAbsoluteValue z + 4 EndAbsoluteValue greater-than 15? 11 less-than z less-than 19 Negative 19 less than z less-than 11 z less-than negative 19 or z greater-than 11 z less-than 19 or z greater-than 11